Física, pregunta formulada por jhonbarc1978ox85fe, hace 11 meses

Cordial saludo: Estoy intentando resolver un ejercicio de física y he visto varios en Brainly exactamente iguales al mío y todos en un punto hacen algo que no entiendo:

0.64= 0.86 Sen(ωt)

7 = 0.86ω Cos(ωt)

Despejando a t de la primera:

t = Sen-1(0.64/0.86)/ω

t= 48.08/ω

¿Cómo llegan al valor de t en ese punto? No entiendo cómo fue a parar 0.64 dentro del paréntesis ni qué es Sen-1(0.64/0.86)/ω. Entiendo que Sen-1 debería ser la inversa del seno pero al hacer inversa del seno de 0.64 entre 0.86 no me da 48.08.

¿Alguien puede explicarme sólo ese paso? Gracias de antemano.


Herminio: Se necesita la ecuación original del problema. La primera ecuación es la de la elongación. La segunda es la derivada de la primera, que es la velocidad. Pero hace falta la ecuación original para ver los valores numéricos
Herminio: Acabo de ver qué es 48,08; el sen(48,08°) es 0,64 / 0,86 = 0,744. Pero el error grande es que el ángulo DEBE expresarse en radianes: 48,08° equivale a 0,839; entonces t = 0,839 / w
jhonbarc1978ox85fe: Ah ok, entonces el mío está bien!! A mpi me daba así.
jhonbarc1978ox85fe: La pregunta original es así:

Una partícula de masa 222 g está unida a un resorte de amplitud 86 cm (Máxima elongación). La partícula oscila sobre una superficie horizontal lisa de tal manera que cuando su estiramiento es de 64 cm, su velocidad de 7 m/s está en dirección +X, tal y como se muestra en la figura.
jhonbarc1978ox85fe: Con base en la anterior información, determine el valor de la constante k de elasticidad del resorte

Respuestas a la pregunta

Contestado por Herminio
2

No son necesarios tantos cálculos.

La velocidad en función de la posición es:

V = ω √(A² - x²), siendo ω la frecuencia angular = √(k/m)

7 m/s = ω √(0,86² - 0,64²) ≅ 0,774 m . ω

Luego ω = 7 m/s / 0,774 m ≅ 9,04 rad/s

k = ω² m = (9.04 rad/s)² .0,222 kg ≅ 18,1 N/m

Saludos Herminio


jhonbarc1978ox85fe: Gracias! Eres un genio!!!
jhonbarc1978ox85fe: Lo que no es los 12,2 rads de dónde salen ahora :(
Herminio: Perdón. Es un error de cálculo, el resultado es 9,04 rad/s y nos queda k = 18,1 N/m
jhonbarc1978ox85fe: Lo siento, pero no entiendo cómo llegas a los 9.04 rad/s. Ese paso no lo pillo.
Otras preguntas